KnowledgeBoat Logo
OPEN IN APP

Question Type 1

Multiple-Choice Questions

Class 10 - ICSE Maths Solved Competency Focused Questions



Multiple Choice Questions (1 Mark Each)

Question 1

A retailer buys an article at its listed price from a wholesaler and sells it to a consumer in the same state after marking up the price by 20%. The list price of the article is ₹ 2500, and the rate of GST is 12%. What is the tax liability of the retailer to the central government?

  1. ₹ 0

  2. ₹ 15

  3. ₹ 30

  4. ₹ 60

Answer

For retailer,

C.P. = ₹ 2500

G.S.T. = 12%

C.G.S.T. = 122\dfrac{12}{2} % = 6%.

C.G.S.T. paid = 6100×2500\dfrac{6}{100} \times 2500 = ₹ 150.

Given,

Retailer sells the article to the consumer in the same state after marking up the price by 20%.

S.P. = ₹ 2500 + 20%

= ₹ 2500 + 20100×2500\dfrac{20}{100} \times 2500

= ₹ 2500 + ₹ 500

= ₹ 3000.

C.G.S.T. = 6%

C.G.S.T. charged = 6100×3000\dfrac{6}{100} \times 3000 = ₹ 180.

Tax liability = C.G.S.T. charged - C.G.S.T. given

= ₹ 180 - ₹ 150

= ₹ 30.

Hence, Option 3 is the correct option.

Question 2

Dev brought an electrical fan which has a marked price of ₹ 800. If the GST on the goods is 7%, then the SGST is :

  1. ₹ 24

  2. ₹ 28

  3. ₹ 56

  4. ₹ 80

Answer

Since, GST rate is 7%, then SGST = 72\dfrac{7}{2} = 3.5%.

M.P. of electrical fan = ₹ 800.

SGST = 3.5% of ₹ 800

= 3.5100×800\dfrac{3.5}{100} \times 800

= ₹ 28.

Hence, Option 2 is the correct option.

Question 3

₹ P is deposited for n number of months in a recurring deposit account which pays interest at the rate of r% per annum. The nature and time of interest calculated is :

  1. compound interest for n number of months

  2. simple interest for n number of months

  3. compound interest for one month

  4. simple interest for one month

Answer

In a recurring deposit account,

By formula,

Interest = P×n(n+1)2×12×r100P \times \dfrac{n(n + 1)}{2 \times 12} \times \dfrac{r}{100}

∴ The nature and time of interest calculated is simple interest for one month.

Hence, Option 4 is the correct option.

Question 4

Anwesha intended to open a Recurring Deposit account of ₹ 1000 per month for 1 year in a Bank, paying a 5% per annum rate of simple interest. The bank reduced the rate to 4% per annum. How much must Anwesha deposit monthly for 1 year so that her interest remains the same?

  1. ₹ 12325

  2. ₹ 1250

  3. ₹ 1200

  4. ₹ 1000

Answer

In first case :

P = ₹ 1000

r = 5%

n = 12 months

Interest = P×n(n+1)2×12×r100P \times \dfrac{n(n + 1)}{2 \times 12} \times \dfrac{r}{100}

=1000×12×(12+1)2×12×5100=1000×12×132×12×120=25×13=325.= 1000 \times \dfrac{12 \times (12 + 1)}{2 \times 12} \times \dfrac{5}{100} \\[1em] = 1000 \times \dfrac{12 \times 13}{2 \times 12} \times \dfrac{1}{20} \\[1em] = 25 \times 13 \\[1em] = ₹ 325.

In second case :

P = ₹ x (Let)

r = 4%

n = 12 months

Interest = ₹ 325

325=x×12×132×12×4100325=x×132×125x=325×25×213x=1625013=₹ 1250.\therefore 325 = x \times \dfrac{12 \times 13}{2 \times 12} \times \dfrac{4}{100} \\[1em] \Rightarrow 325 = x \times \dfrac{13}{2} \times \dfrac{1}{25}\\[1em] \Rightarrow x = \dfrac{325 \times 25 \times 2}{13} \\[1em] \Rightarrow x = \dfrac{16250}{13} = \text{₹ 1250.}

Hence, Option 2 is the correct option.

Question 5

Mr. Das invests in ₹ 100, 12% shares of Company A available at ₹ 60 each. Mr. Singh invests in ₹ 50, 16% shares of Company B available at ₹ 40 each. Use this information to state which of the following statements is true.

  1. The rate of return for Mr. Das is 12%

  2. The rate of return for Mr. Singh is 10%

  3. Both Mr. Das and Mr. Singh have the same rate of return of 10%

  4. Both Mr. Das and Mr. Singh have the same rate of return of 20%

Answer

For Mr. Das,

Face value of each share = ₹ 100

Market value of each share = ₹ 60

Dividend per share = 12% of ₹ 100 = ₹ 12.

Rate of return = Dividend per shareM.V. of each share×100=1260×100\dfrac{\text{Dividend per share}}{\text{M.V. of each share}} \times 100 = \dfrac{12}{60} \times 100 = 20%.

For Mr. Singh,

Face value of each share = ₹ 50

Market value of each share = ₹ 40

Dividend per share = 16% of ₹ 50 = ₹ 8.

Rate of return = Dividend per shareM.V. of each share×100=840×100\dfrac{\text{Dividend per share}}{\text{M.V. of each share}} \times 100 = \dfrac{8}{40} \times 100 = 20%.

∴ Both Mr. Das and Mr. Singh have the same rate of return of 20%.

Hence, Option 4 is the correct option.

Question 6

Amit invested a certain sum of money in ₹ 100 shares, paying a 7.5% dividend. The rate of return on his investment is 10%. The money invested by Amit to purchase 10 shares is :

  1. ₹ 250

  2. ₹ 750

  3. ₹ 900

  4. ₹ 1100

Answer

Let ₹ P be the price per share.

Amit brought 10 shares, so total investment = ₹ 10P

Dividend = 7.5%

Dividend per share = 7.5100×100\dfrac{7.5}{100} \times 100 = ₹ 7.5

Total dividend = ₹ 7.5 × 10 = ₹ 75.

Rate of return = 10%

By formula,

Rate of return × Investment = Dividend

1010100×10P=75100P100=75P=75.\Rightarrow 10% \times 10P = 75 \\[1em] \Rightarrow \dfrac{10}{100} \times 10P = 75 \\[1em] \Rightarrow \dfrac{100P}{100} = 75 \\[1em] \Rightarrow P = ₹ 75.

Total investment = 10P = 10 × ₹75 = ₹750.

Hence, Option 2 is the correct option.

Question 7

If -3 ≤ -4x + 5 and x ∈ W, then the solution set is :

  1. {.......-3, -2, -1, 0, 1, 2, 3, .......}

  2. {1, 2}

  3. {0, 1, 2}

  4. {2, 3, 4, 5}

Answer

Solving,

⇒ -3 ≤ -4x + 5

⇒ 4x ≤ 5 + 3

⇒ 4x ≤ 8

⇒ x ≤ 84\dfrac{8}{4}

⇒ x ≤ 2.

Since, x ≤ 2 and x ∈ W.

∴ Solution set = {0, 1, 2}.

Hence, Option 3 is the correct option.

Question 8

If -4x > 8y; then

  1. x > 2y

  2. x > -2y

  3. x < -2y

  4. x < 2y

Answer

Solving,

⇒ -4x > 8y

⇒ 4x < -8y

⇒ x < 8y4-\dfrac{8y}{4}

⇒ x < -2y.

Hence, Option 3 is the correct option.

Question 9

The value/s of 'k' for which the quadratic equation 2x2 - kx + k = 0 has equal roots is (are) :

  1. 0 only

  2. 4, 0

  3. 8 only

  4. 0, 8

Answer

Given,

Equation : 2x2 - kx + k = 0

a = 2, b = -k and c = k.

For equal roots,

⇒ Discriminant (D) = 0

⇒ b2 - 4ac = 0

⇒ (-k)2 - 4 × 2 × k = 0

⇒ k2 - 8k = 0

⇒ k(k - 8) = 0

⇒ k = 0 or k - 8 = 0

⇒ k = 0 or k = 8.

Hence, Option 4 is the correct option.

Question 10

If x = -2 is one of the solutions of the quadratic equation x2 + 3a - x = 0, then the value of 'a' is :

  1. -8

  2. -2

  3. 13-\dfrac{1}{3}

  4. 13\dfrac{1}{3}

Answer

Given,

x = -2 is one of the solutions of the quadratic equation x2 + 3a - x = 0.

∴ (-2)2 + 3a - (-2) = 0

⇒ 4 + 3a + 2 = 0

⇒ 3a + 6 = 0

⇒ 3a = -6

⇒ a = 63-\dfrac{6}{3} = -2.

Hence, Option 2 is the correct option.

Question 11

In solving a quadratic equation, one of the values of the variables x is 233.356. The solution rounded to two significant figures is :

  1. 233.36

  2. 233.35

  3. 233.3

  4. 230

Answer

x = 233.356

On rounding off to two significant figures

x = 230.

Hence, Option 4 is the correct option.

Question 12

In the adjoining diagram, AB = x cm, BC = y cm and x - y = 7 cm. Area of △ ABC = 30 cm2. The length of AC is :

  1. 10 cm

  2. 12 cm

  3. 13 cm

  4. 15 cm

In the adjoining diagram, AB = x cm, BC = y cm and x - y = 7 cm. Area of △ ABC = 30 cm2. The length of AC is : Maths Competency Focused Practice Questions Class 10 Solutions.

Answer

By formula,

Area of triangle = 12× base×height\dfrac{1}{2} \times \text{ base} \times \text{height}

⇒ 30 = 12×BC×AB\dfrac{1}{2} \times BC \times AB

⇒ 30 = 12×y×x\dfrac{1}{2} \times y \times x

⇒ 30 = xy2\dfrac{xy}{2}

⇒ xy = 60 ........(1)

Given,

⇒ x - y = 7

⇒ x = 7 + y ........(2)

Substituting value of x from equation (2) in (1), we get :

⇒ (7 + y)y = 60

⇒ 7y + y2 = 60

⇒ y2 + 7y - 60 = 0

⇒ y2 + 12y - 5y - 60 = 0

⇒ y(y + 12) - 5(y + 12) = 0

⇒ (y - 5)(y + 12) = 0

⇒ y - 5 = 0 or y + 12 = 0

⇒ y = 5 or y = -12.

Since, side cannot be negative,

∴ y = 5 cm.

Substituting value of y in equation (2), we get :

⇒ x = 7 + y = 7 + 5 = 12 cm.

In right angle triangle ABC,

By pythagoras theorem,

⇒ AC2 = AB2 + BC2

⇒ AC2 = x2 + y2

⇒ AC2 = 122 + 52

⇒ AC2 = 144 + 25

⇒ AC2 = 169

⇒ AC = 169\sqrt{169} = 13 cm.

Hence, Option 3 is the correct option.

Question 13

If p, q, and r are in continued proportion, then :

  1. p : q = p : r

  2. q : r = p2 : q2

  3. p : q2 = r : p2

  4. p : r = p2 : q2

Answer

Since, p, q, and r are in continued proportion.

pq=qrq2=pr ........(1)\therefore \dfrac{p}{q} = \dfrac{q}{r} \\[1em] \Rightarrow q^2 = pr \text{ ........(1)}

Solving,

p : r = p2 : q2

pr=p2q2q2=p2×rpq2=pr .........(2)\Rightarrow \dfrac{p}{r} = \dfrac{p^2}{q^2} \\[1em] \Rightarrow q^2 = \dfrac{p^2 \times r}{p} \\[1em] \Rightarrow q^2 = pr \text{ .........(2)}

Since, equation (1) and (2) are equal.

Hence, Option 4 is the correct option.

Question 14

The ratio of diameter to height of a Borosil cylindrical glass is 3 : 5. If the actual diameter of the glass is 6 cm, then the curved surface area of the glass is :

  1. 120π

  2. 60π

  3. 30π

  4. 18π

Answer

Given,

Ratio of diameter to height of a Borosil cylindrical glass is 3 : 5.

dh=356h=35h=6×53h=10 cm.\Rightarrow \dfrac{d}{h} = \dfrac{3}{5} \\[1em] \Rightarrow \dfrac{6}{h} = \dfrac{3}{5} \\[1em] \Rightarrow h = \dfrac{6 \times 5}{3} \\[1em] \Rightarrow h = 10 \text{ cm}.

Radius = Diameter2=62\dfrac{\text{Diameter}}{2} = \dfrac{6}{2} = 3 cm.

By formula,

Curved surface area of glass = 2πrh

= 2π × 3 × 10

= 60π cm2.

Hence, Option 2 is the correct option.

Question 15

If the polynomial 2x3 + 3x2 - 2x - 3 is completely divisible by (2x + a), and the quotient is equal to (x2 - 1), then one of the values of a is :

  1. -3

  2. -1

  3. 1

  4. 3

Answer

Given,

The polynomial 2x3 + 3x2 - 2x - 3 is completely divisible by (2x + a) and quotient is equal to (x2 - 1).

∴ 2x3 + 3x2 - 2x - 3 = (2x + a)(x2 - 1)

⇒ 2x3 + 3x2 - 2x - 3 = 2x3 - 2x + ax2 - a

⇒ 2x3 - 2x3 + 3x2 - 2x + 2x - 3 = ax2 - a

⇒ 3x2 - 3 = ax2 - a

From above equation,

a = 3.

Hence, Option 4 is the correct option.

Question 16

A polynomial in x is x3 + 5x2 - kx - 24. Which of the following is a factor of the given polynomial so that the value of k is 2?

  1. (x + 2)

  2. (x - 3)

  3. (x + 4)

  4. (x - 4)

Answer

By factor theorem,

(x - a) is a factor of f(x), if f(a) = 0.

Given,

Polynomial = x3 + 5x2 - kx - 24

If k = 2, then :

Polynomial = x3 + 5x2 - 2x - 24.

Dividing polynomial by (x + 4) or substituting -4 in polynomial, we get :

⇒ (-4)3 + 5(-4)2 - 2(-4) - 24

⇒ -64 + 5(16) + 8 - 24

⇒ -64 + 80 + 8 - 24

⇒ 88 - 88

⇒ 0.

Since, on substituting -4 in polynomial, we get remainder = 0.

∴ (x + 4) is the factor of x3 + 5x2 - kx - 24, when k = 2.

Hence, Option 3 is the correct option.

Question 17

If A = [ab] and [cd]\begin{bmatrix*}[r] a & b \end{bmatrix*} \text{ and } \begin{bmatrix*}[r] c \\ d \end{bmatrix*}, then :

  1. only matrix AB is possible

  2. only matrix BA is possible

  3. both matrices AB and BA are possible

  4. both matrices AB and BA are possible, AB = BA

Answer

Order of matrix A = 1 × 2

Order of matrix B = 2 × 1

Since, no. of columns in A is equal to the no. of rows in B and no. of columns in B is equal to the no. of rows in A.

∴ AB and BA are possible.

AB=[a×c+b×d]=[ac+bd].BA=[c×ac×bd×ad×b]=[cacbdadb].AB = \begin{bmatrix*}[r] a \times c + b \times d \end{bmatrix*} \\[1em] = \begin{bmatrix*}[r] ac + bd \end{bmatrix*}. \\[1em] BA = \begin{bmatrix*}[r] c \times a & c \times b \\ d \times a & d \times b \end{bmatrix*} \\[1em] = \begin{bmatrix*}[r] ca & cb \\ da & db \end{bmatrix*}.

∴ AB ≠ BA.

Hence, Option 3 is the correct option.

Question 18

Matrix A = [694k] such that A2=[0000]\begin{bmatrix*}[r] 6 & 9 \\ -4 & k \end{bmatrix*} \text{ such that } A^2 = \begin{bmatrix*}[r] 0 & 0 \\ 0 & 0 \end{bmatrix*}. Then k is :

  1. 6

  2. -6

  3. 36

  4. ±6

Answer

Given, A2=[0000]A^2 = \begin{bmatrix*}[r] 0 & 0 \\ 0 & 0 \end{bmatrix*}.

[694k][694k]=[0000][6×6+9×(4)6×9+9×k4×6+k×(4)(4)×9+k×k]=[0000][36+(36)54+9k244k36+k2]=[0000][054+9k244k36+k2]=[0000]54+9k=09k=54k=549=6.\therefore \begin{bmatrix*}[r] 6 & 9 \\ -4 & k \end{bmatrix*}\begin{bmatrix*}[r] 6 & 9 \\ -4 & k \end{bmatrix*} = \begin{bmatrix*}[r] 0 & 0 \\ 0 & 0 \end{bmatrix*} \\[1em] \Rightarrow \begin{bmatrix*}[r] 6 \times 6 + 9 \times (-4) & 6 \times 9 + 9 \times k \\ -4 \times 6 + k \times (-4) & (-4) \times 9 + k \times k \end{bmatrix*} = \begin{bmatrix*}[r] 0 & 0 \\ 0 & 0 \end{bmatrix*} \\[1em] \Rightarrow \begin{bmatrix*}[r] 36 + (-36) & 54 + 9k \\ -24 - 4k & -36 + k^2 \end{bmatrix*} = \begin{bmatrix*}[r] 0 & 0 \\ 0 & 0 \end{bmatrix*} \\[1em] \Rightarrow \begin{bmatrix*}[r] 0 & 54 + 9k \\ -24 - 4k & -36 + k^2 \end{bmatrix*} = \begin{bmatrix*}[r] 0 & 0 \\ 0 & 0 \end{bmatrix*} \\[1em] \Rightarrow 54 + 9k = 0 \\[1em] \Rightarrow 9k = -54 \\[1em] \Rightarrow k = -\dfrac{54}{9} = -6.

Hence, Option 2 is the correct option.

Question 19

If the sum of n terms of an arithmetic progression Sn = n2 - n, then the third term of the series is :

  1. 2

  2. 4

  3. 6

  4. 9

Answer

Given,

Sum of n terms of an arithmetic progression Sn = n2 - n.

S1 = 12 - 1 = 0,

S2 = 22 - 2 = 4 - 2 = 2,

S3 = 32 - 3 = 9 - 3 = 6.

Sum upto first term = First term = 0.

Given, sum upto 2 terms = 2 and first term = 0, second term = 2.

Sum upto third term = 6

∴ First term + Second term + Third term = 6

⇒ 0 + 2 + Third term = 6

⇒ Third term = 6 - 2 = 4.

Hence, Option 2 is the correct option.

Question 20

Which of the following is not a geometric progression ?

  1. 13,1,3,9\dfrac{1}{3}, 1, 3, 9

  2. 15,15,15,15\dfrac{1}{5}, \dfrac{1}{5}, \dfrac{1}{5}, \dfrac{1}{5}

  3. -2, 4, -8, 16

  4. 2, 0, 4, 0, 8, 0

Answer

In the series,

2, 0, 4, 0, 8, 0

0240\dfrac{0}{2} \ne \dfrac{4}{0}

∴ 2, 0, 4, 0, 8, 0 is not a G.P.

Hence, Option 4 is the correct option.

Question 21

In the adjoining diagram, G is the centroid of △ ABC. A(3, -3), B(2, -6), C(x, y) and G(5, -5). The coordinates of point D are :

  1. (2, -6)

  2. (3, -6)

  3. (6, -6)

  4. (10, -6)

In the adjoining diagram, G is the centroid of △ ABC. A(3, -3), B(2, -6), C(x, y) and G(5, -5). The coordinates of point D are : Maths Competency Focused Practice Questions Class 10 Solutions.

Answer

By formula,

Centroid of triangle = (x1+x2+x33,y1+y2+y33)\Big(\dfrac{x_1 + x_2 + x_3}{3}, \dfrac{y_1 + y_2 + y_3}{3}\Big)

(5,5)=(3+2+x3,(3)+(6)+y3)(5,5)=(x+53,y93)x+53=5 and y93=5x+5=15 and y9=15x=155=10 and y=15+9=6.\therefore (5, -5) = \Big(\dfrac{3 + 2 + x}{3}, \dfrac{(-3) + (-6) + y}{3}\Big) \\[1em] \Rightarrow (5, -5) = \Big(\dfrac{x + 5}{3}, \dfrac{y - 9}{3}\Big) \\[1em] \Rightarrow \dfrac{x + 5}{3} = 5 \text{ and } \dfrac{y - 9}{3} = -5 \\[1em] \Rightarrow x + 5 = 15 \text{ and } y - 9 = -15 \\[1em] \Rightarrow x = 15 - 5 = 10 \text{ and } y = -15 + 9 = -6.

C(x, y) = (10, -6).

Since, centroid is the point of intersection of all the three medians of a triangle.

∴ AD is the median.

∴ D is mid-point of BC.

D=(2+102,(6)+(6)2)=(122,122)=(6,6).D = \Big(\dfrac{2 + 10}{2}, \dfrac{(-6) + (-6)}{2}\Big) \\[1em] = \Big(\dfrac{12}{2}, \dfrac{-12}{2}\Big) \\[1em] = (6, -6).

Hence, Option 3 is the correct option.

Question 22

In the given diagram, O is the origin and P is the mid-point of AB. The equation of OP is :

  1. y = x

  2. 2y = x

  3. y = 2x

  4. y = -x

In the given diagram, O is the origin and P is the mid-point of AB. The equation of OP is : Maths Competency Focused Practice Questions Class 10 Solutions.

Answer

From graph,

A = (4, 0) and B = (0, 2).

Given,

P is the mid-point of AB.

P = (4+02,0+22)=(42,22)\Big(\dfrac{4 + 0}{2}, \dfrac{0 + 2}{2}\Big) = \Big(\dfrac{4}{2}, \dfrac{2}{2}\Big) = (2, 1).

By two-point form,

⇒ y - y1 = y2y1x2x1(xx1)\dfrac{y_2 - y_1}{x_2 - x_1}(x - x_1)

⇒ y - 1 = 1020(x2)\dfrac{1 - 0}{2 - 0}(x - 2)

⇒ y - 1 = 12(x2)\dfrac{1}{2}(x - 2)

⇒ 2(y - 1) = x - 2

⇒ 2y - 2 = x - 2

⇒ 2y = x - 2 + 2

⇒ 2y = x.

Hence, Option 2 is the correct option.

Question 23

In the given figure line l1 is a parallel to line l2. If line l3 is perpendicular to line l1, then the slopes of lines l2 and l3 respectively are :

  1. 1, 1

  2. -1, -1

  3. 1, -1

  4. -1, 1

In the given figure line l1 is a parallel to line l2. If line l3 is perpendicular to line l1, then the slopes of lines l2 and l3 respectively are : Maths Competency Focused Practice Questions Class 10 Solutions.

Answer

Slope of line l1 = tan 45° = 1.

We know that,

Slope of parallel lines are equal.

Slope of line l2 = Slope of line l1 = 1.

We know that,

Product of slope of perpendicular lines is equal to -1.

⇒ Slope of line l2 × Slope of line l3 = -1

⇒ 1 × Slope of line l3 = -1

⇒ Slope of line l3 = -1.

Hence, Option 3 is the correct option.

Question 24

Which of the following lines cut the positive x-axis and positive y-axis at equal distances form the origin?

  1. 3x + 3y = 6

  2. 5x + 10y = 10

  3. -x + y = 1

  4. 10x + 5y = 5

Answer

Substituting x = 0 in first equation,

⇒ 3(0) + 3y = 6

⇒ 3y = 6

⇒ y = 63\dfrac{6}{3}

⇒ y = 2.

The line touches y-axis at point (0, 2).

Substituting y = 0 in first equation,

⇒ 3x + 3(0) = 6

⇒ 3x = 6

⇒ x = 63\dfrac{6}{3}

⇒ x = 2.

The line touches y-axis at point (2, 0).

∴ Line 3x + 3y = 6 cuts positive x-axis and positive y-axis at equal distance i.e. 2 units form the origin.

Hence, Option 1 is the correct option.

Question 25

In the given diagram (not draw to scale), railway stations A, B, C, P and Q are connected by straight tracks. Track PQ is parallel to BC. The time taken by a train travelling at 90 km/hr to reach B from A by the shortest route is :

  1. 8 minutes

  2. 12 minutes

  3. 16.8 minutes

  4. 20 minutes

In the given diagram (not draw to scale), railway stations A, B, C, P and Q are connected by straight tracks. Track PQ is parallel to BC. The time taken by a train travelling at 90 km/hr to reach B from A by the shortest route is : Maths Competency Focused Practice Questions Class 10 Solutions.

Answer

In △ APQ and △ ABC,

⇒ ∠PAQ = ∠BAC (Common angle)

⇒ ∠APQ = ∠ABC (Corresponding angles are equal)

∴ △ APQ ~ △ ABC (By A.A. axiom)

From figure,

Let AP = x km.

We know that,

Corresponding sides of similar triangle are proportional.

APAB=PQBCxAP+PB=2050xx+18=255x=2(x+18)5x=2x+365x2x=363x=36x=363x=12.\Rightarrow \dfrac{AP}{AB} = \dfrac{PQ}{BC} \\[1em] \Rightarrow \dfrac{x}{AP + PB} = \dfrac{20}{50} \\[1em] \Rightarrow \dfrac{x}{x + 18} = \dfrac{2}{5} \\[1em] \Rightarrow 5x = 2(x + 18) \\[1em] \Rightarrow 5x = 2x + 36 \\[1em] \Rightarrow 5x - 2x = 36 \\[1em] \Rightarrow 3x = 36 \\[1em] \Rightarrow x = \dfrac{36}{3} \\[1em] \Rightarrow x = 12.

AB = AP + BP = 12 + 18 = 30 km.

Time = DistanceSpeed=AB90=3090=13 hr=13×60\dfrac{\text{Distance}}{\text{Speed}} = \dfrac{AB}{90} = \dfrac{30}{90} = \dfrac{1}{3} \text{ hr} = \dfrac{1}{3} \times 60 = 20 minutes.

Hence, Option 4 is the correct option.

Question 26

In the given diagram, △ ABC and △ DEF (not drawn to scale) are such that ∠C = ∠F and ABDE=BCEF\dfrac{AB}{DE} = \dfrac{BC}{EF}, then

  1. △ ABC ~ △ DEF

  2. △ BCA ~ △ DEF

  3. △ CAB ~ △ DEF

  4. the similarity of given triangles cannot be determined.

In the given diagram, △ ABC and △ DEF (not drawn to scale) are such that ∠C = ∠F and ABDE = BCEF, then : Maths Competency Focused Practice Questions Class 10 Solutions.

Answer

Given,

⇒ ∠C = ∠F

Since, the sides containing the angle may or may not be proportional,

i.e., we don't know if ACDF\dfrac{AC}{DF} and BCEF\dfrac{BC}{EF} are equal or not equal.

∴ Similarity of given triangles cannot be determined.

Hence, Option 4 is the correct option.

Question 27

In the adjoining diagram, ST is not parallel to PQ. The necessary and sufficient conditions for △ PQR ~ △ TSR is :

  1. ∠PQR = ∠STR

  2. ∠QPR = ∠TSR

  3. ∠PQR = ∠TSR

  4. ∠PRQ = ∠RST

In the adjoining diagram, ST is not parallel to PQ. The necessary and sufficient conditions for △ PQR ~ △ TSR is : Maths Competency Focused Practice Questions Class 10 Solutions.

Answer

Given,

△ PQR ~ △ TSR.

From figure,

⇒ ∠PRQ = ∠SRT (Common angle)

The order of vertices of two similar triangles are written in such a way that the corresponding vertices occupy the same position.

∴ ∠PQR = ∠TSR

∴ △ PQR ~ △ TSR (By A.A. axiom)

Hence, Option 3 is the correct option.

Question 28

The scale factor of a picture and the actual height of Sonia is 20 cm : 1.6 m. If her height in the picture is 18 cm, then her actual height is :

  1. 14.4 m

  2. 2.25 m

  3. 1.78 m

  4. 1.44 m

Answer

Scale factor of a picture and the actual height = 20 cm : 1.6 m = 20 cm : 160 cm

Height in pictureActual height=20 cm160 cm18Actual height=20 cm160 cmActual height=18×16020=144 cm=1.44 m.\therefore \dfrac{\text{Height in picture}}{\text{Actual height}} = \dfrac{\text{20 cm}}{\text{160 cm}} \\[1em] \Rightarrow \dfrac{18}{\text{Actual height}} = \dfrac{\text{20 cm}}{\text{160 cm}} \\[1em] \Rightarrow \text{Actual height} = \dfrac{18 \times 160}{20} = \text{144 cm} = 1.44 \text{ m}.

Hence, Option 4 is the correct option.

Question 29

In the adjoining figure, O is the center of the circle, and a semicircle is drawn on OA as the diameter. ∠APQ = 20°. The degree measure of ∠OAQ is :

  1. 25°

  2. 40°

  3. 50°

  4. 65°

In the adjoining figure, O is the center of the circle, and a semicircle is drawn on OA as the diameter. ∠APQ = 20°. The degree measure of ∠OAQ is : Maths Competency Focused Practice Questions Class 10 Solutions.

Answer

We know that,

Angle in a semi-circle is a right angle.

∴ ∠OQA = 90°

In △QAP,

⇒ ∠PQA + ∠QAP + ∠APQ = 180°

⇒ 90° + ∠QAP + 20° = 180°

⇒ ∠QAP = 180° - 90° - 20° = 70°.

In △OPA,

⇒ OA = OP (Radii of same circle)

⇒ ∠OAP = ∠OPA (Angle opposite to equal sides are equal)

⇒ ∠OAP = 20°.

From figure,

⇒ ∠OAQ = ∠QAP - ∠OAP = 70° - 20° = 50°.

Hence, Option 3 is the correct option.

Question 30

In the given diagram, O is the center of the circle, and PQ is a tangent at A. If ∠ABC = 50°, then values of x, y and z respectively are :

  1. 50°, 100°, 40°

  2. 50°, 50°, 65°

  3. 40°, 80°, 50°

  4. 50°, 25°, 78°

In the given diagram, O is the center of the circle, and PQ is a tangent at A. If ∠ABC = 50°, then values of x, y and z respectively are : Maths Competency Focused Practice Questions Class 10 Solutions.

Answer

We know that,

The angle subtended by an arc at the centre is double the angle subtended by it at any point on the remaining part of the circle.

∴ ∠AOC (y) = 2∠ABC = 2 × 50° = 100°.

In △ AOC,

⇒ OA = OC (Radii of same circle)

⇒ ∠OAC = ∠OCA = z (Angle opposite to equal sides are equal)

By angle sum property of triangle,

⇒ ∠OAC + ∠OCA + ∠AOC = 180°

⇒ z + z + 100° = 180°

⇒ 2z = 180° - 100°

⇒ 2z = 80°

⇒ z = 80°2\dfrac{80°}{2} = 40°.

We know that,

The tangent at any point of a circle is perpendicular to the radius through the point of contact.

⇒ ∠CAQ (x) = ∠OAQ - ∠OAC = 90° - 40° = 50°.

Hence, Option 1 is the correct option.

Question 31

In the given figure, PT and QT are tangents to a circle such that ∠TPS = 45° and ∠TQS = 30°. Then, the value of x is :

  1. 30°

  2. 45°

  3. 75°

  4. 105°

In the given figure, PT and QT are tangents to a circle such that ∠TPS = 45° and ∠TQS = 30°. Then, the value of x is : Maths Competency Focused Practice Questions Class 10 Solutions.

Answer

We know that,

The angle between a tangent and a chord through point of contact is equal to an angle in the alternate segment.

∴ ∠SQP = ∠SPT = 45° and ∠SPQ = ∠SQT = 30°.

In △ SQP,

⇒ ∠SQP + ∠SPQ + ∠QSP = 180°

⇒ 45° + 30° + ∠QSP = 180°

⇒ 75° + ∠QSP = 180°

⇒ ∠QSP (x) = 180° - 75° = 105°.

Hence, Option 4 is the correct option.

Question 32

A cylindrical metallic wire is stretched to double its length. Which of the following will NOT change for the wire after stretching?

  1. Its curved surface area

  2. Its total surface area

  3. Its volume

  4. Its radius

Answer

On changing the shape of a container, its volume remains same.

Hence, Option 3 is the correct option.

Question 33

A right circular cone has the radius of the base equal to the height of the cone. If the volume of the cone is 9702 cu. cm, then the diameter of the base of the cone is :

  1. 21 cm

  2. 42 cm

  3. 21721\sqrt{7} cm

  4. 272\sqrt{7} cm

[Use π=227]\Big[\text{Use } \pi = \dfrac{22}{7}\Big]

Answer

Given,

Height of cone (h) = Radius of cone (r) = a cm (let)

A right circular cone has the radius of the base equal to the height of the cone. If the volume of the cone is 9702 cu. cm, then the diameter of the base of the cone is : Maths Competency Focused Practice Questions Class 10 Solutions.

Given,

Volume = 9702 cm3

13πr2h=970213×227×a2×a=9702a3=9702×7×322a3=441×21a3=9261a=92613=21 cm.\therefore \dfrac{1}{3}πr^2h = 9702 \\[1em] \Rightarrow \dfrac{1}{3} \times \dfrac{22}{7} \times a^2 \times a = 9702 \\[1em] \Rightarrow a^3 = \dfrac{9702 \times 7 \times 3}{22} \\[1em] \Rightarrow a^3 = 441 \times 21 \\[1em] \Rightarrow a^3 = 9261 \\[1em] \Rightarrow a = \sqrt[3]{9261} = 21 \text{ cm}.

Diameter = 2 × radius = 2 × 21 = 42 cm.

Hence, Option 2 is the correct option.

Question 34

A solid sphere with a radius of 4 cm is cut into 4 identical pieces by two mutually perpendicular planes passing through its center. Find the total surface area of one-quarter piece.

  1. 24π

  2. 32π

  3. 48π

  4. 64π

A solid sphere with a radius of 4 cm is cut into 4 identical pieces by two mutually perpendicular planes passing through its center. Find the total surface area of one-quarter piece. Maths Competency Focused Practice Questions Class 10 Solutions.

Answer

Total surface area of semi-hemisphere = 2πr2

= 2π × 42

= 2π × 16

= 32π.

Hence, Option 2 is the correct option.

Question 35

Two identical solid hemispheres are kept in contact to form a sphere. The ratio of the total surface areas of two hemispheres to the surface area of the sphere formed is :

  1. 1 : 1

  2. 3 : 2

  3. 2 : 3

  4. 2 : 1

Two identical solid hemispheres are kept in contact to form a sphere. The ratio of the total surface areas of two hemispheres to the surface area of the sphere formed is : Maths Competency Focused Practice Questions Class 10 Solutions.

Answer

Let radius of hemisphere be r.

Total surface area of hemisphere = 3πr2

Total surface area of two hemisphere = 2 × 3πr2 = 6πr2.

Total surface area of sphere = 4πr2

Total surface area of two hemisphere : Total surface area of sphere = 6πr2 : 4πr2

= 6 : 4

= 3 : 2.

Hence, Option 2 is the correct option.

Question 36

cosec2 θ + sec2 θ is equal to :

  1. tan2 θ + cot2 θ

  2. cot θ + tan θ

  3. (cot θ + tan θ)2

  4. 1

Answer

Solving,

    cosec2 θ + sec2 θ

= 1 + cot2 θ + 1 + tan2 θ

= cot2 θ + tan2 θ + 2

= (cot θ + tan θ)2

Hence, Option 3 is the correct option.

Question 37

Given a = 3 sec2 θ and b = 3 tan2 θ - 2. The value of (a - b) is :

  1. 1

  2. 2

  3. 3

  4. 5

Answer

    a - b = 3 sec2 θ - (3 tan2 θ - 2)

= 3 sec2 θ - 3 tan2 θ + 2

= 3(sec2 θ - tan2 θ) + 2

= 3(1) + 2

= 3 + 2

= 5.

Hence, Option 4 is the correct option.

Question 38

At a certain time of day, the ratio of the height of the pole to the length of its shadow is 1 : 3\sqrt{3}, then the angle of elevation of the sun at that time of the day is :

  1. 30°

  2. 45°

  3. 60°

  4. 90°

Answer

Let AB be the pole and BC be the shadow and angle of elevation of Sun be θ.

At a certain time of day, the ratio of the height of the pole to the length of its shadow is 1 : 3, then the angle of elevation of the sun at that time of the day is : Maths Competency Focused Practice Questions Class 10 Solutions.

From figure,

⇒ tan θ = ABBC\dfrac{AB}{BC}

⇒ tan θ = x3x\dfrac{x}{\sqrt{3}x}

⇒ tan θ = 13\dfrac{1}{\sqrt{3}}

⇒ tan θ = tan 30°

⇒ θ = 30°.

Hence, Option 1 is the correct option.

Question 39

A man standing on a ship approaching the port towards the lighthouse is observing the top of the lighthouse. In 10 minutes, the angle of elevation of the top of the lighthouse changes from α to β. Then :

  1. α > β

  2. α < β

  3. α = β

  4. α ≤ β

Answer

Let A be the top of the lighthouse, C the initial position of ship and D be the position after 10 minutes.

A man standing on a ship approaching the port towards the lighthouse is observing the top of the lighthouse. In 10 minutes, the angle of elevation of the top of the lighthouse changes from α to β. Then : Maths Competency Focused Practice Questions Class 10 Solutions.

From figure,

tan α = ABBC\dfrac{AB}{BC}

tan β = ABBD\dfrac{AB}{BD}

Since, BC is greater than BD.

∴ tan α < tan β

⇒ α < β.

Hence, Option 2 is the correct option.

Question 40

Assertion (A) : The difference in class marks of the modal class and the median class of the following frequency distribution table is 0.

Class intervalFrequency
20-301
30-403
40-502
50-606
60-704

Reason (R) : Modal class and median class are always the same for a given frequency distribution.

  1. Both A and R are correct, and R is the correct explanation for A.

  2. Both A and R are correct, and R is not the correct explanation for A.

  3. A is true, but R is false.

  4. Both A and R are true.

Answer

Cumulative frequency distribution table :

Class intervalClass markFrequencyCumulative frequency
20-302511
30-403534
40-504526
50-6055612
60-7065416

Median = 162\dfrac{16}{2} = 8th term.

The 8th term lies in the class 50-60.

∴ Median class = 50-60

Also, frequency of class 50-60 is highest.

∴ Modal class = 50-60.

∴ Assertion (A) is true.

Modal class and median class are not always the same for a given frequency distribution.

∴ Reason (R) is false.

Hence, Option 3 is the correct option.

Question 41

Assertion (A) : For a collection of 11 arrayed data, the median is the middle number.

Reason (R) : For the data 5, 9, 7, 13, 10, 11, 10, the median is 13.

  1. Both A and R are correct, and R is the correct explanation for A.

  2. Both A and R are correct, and R is not the correct explanation for A.

  3. A is true, but R is false.

  4. Both A and R are true.

Answer

For 11 arrayed data.

Median = n+12\dfrac{n + 1}{2} th term

= 11+12\dfrac{11 + 1}{2}

= 6th term, which will be the middle term.

∴ For a collection of 11 arrayed data, the median is the middle number.

∴ Assertion (A) is true.

Numbers = 5, 9, 7, 13, 10, 11, 10

Arranging in ascending order, we get :

5, 7, 9, 10, 10, 11, 13.

n = 7, which is odd.

Median = n+12=7+12=82\dfrac{n + 1}{2} = \dfrac{7 + 1}{2} = \dfrac{8}{2} = 4th term = 10.

∴ Reason (R) is false.

Hence, Option 3 is the correct option.

Question 42

Ankit has the option of investing in company A, where 7%, ₹ 100 shares are available at ₹ 120 or in company B, where 8%, ₹ 1000 shares are available at ₹ 1620.

Assertion (A) : Investment in Company A is better than Company B.

Reason (R) : The rate of income in Company A is better than in Company B.

  1. Both A and R are correct, and R is the correct explanation for A.

  2. Both A and R are correct, and R is not the correct explanation for A.

  3. A is false, but R is true.

  4. Both A and R are false.

Answer

In company A,

N.V. = ₹ 100

M.V. = ₹ 120

Dividend = 7% = 7100×100\dfrac{7}{100} \times 100 = ₹ 7

∴ Investment = ₹ 120 and income = ₹ 7.

Income on ₹ 1 = 7120\dfrac{7}{120} = ₹ 0.0583

In company B,

N.V. = ₹ 1000

M.V. = ₹ 1620

Dividend = 8% = 8100×1000\dfrac{8}{100} \times 1000 = ₹ 80

∴ Investment = ₹ 1620 and income = ₹ 80.

Income on ₹ 1 = 801620\dfrac{80}{1620} = ₹ 0.0494

Since, rate of income is greater in company A.

∴ Assertion and Reason both are true and Reason is the correct explanation of Assertion.

Hence, Option 1 is the correct explanation.

Question 43

Assertion (A) : x3 + 2x2 - x - 2 is a polynomial of degree 3.

Reason (R) : x + 2 is a factor of the polynomial.

  1. Both A and R are correct, and R is the correct explanation for A.

  2. Both A and R are correct, and R is not the correct explanation for A.

  3. A is true, but R is false.

  4. Both A and R are true.

Answer

x3 + 2x2 - x - 2 is a polynomial of degree 3.

By factor theorem,

(x - a) is a factor of f(x) if f(a) = 0.

x + 2 = 0

x = -2

Substituting x = -2 in x3 + 2x2 - x - 2, we get :

⇒ (-2)3 + 2(-2)2 - (-2) - 2

⇒ -8 + 2(4) + 2 - 2

⇒ -8 + 8 + 2 - 2

⇒ 0.

Hence, Option 4 is the correct option.

Question 44

Assertion (A) : The point (-2, 8) is invariant under reflection in line x = -2

Reason (R) : If a point has its x-coordinate 0, it is invariant under reflection in both axes.

  1. Both A and R are correct, and R is the correct explanation for A.

  2. Both A and R are correct, and R is not the correct explanation for A.

  3. A is true, but R is false.

  4. Both A and R are true.

Answer

Point (-2, 8) lies on the line x = -2.

The point which lies on a line is invariant under reflection in the same line.

∴ (-2, 8) is invariant under reflection in line x = -2.

∴ Assertion (A) is true.

If a point has x-coordinate equal to 0 it means it lies on y-axis.

∴ It will be invariant under reflection in y-axis.

∴ Reason (R) is false.

Hence, Option 3 is the correct option.

Question 45

When a die is cast with numbering on its faces, as shown, the ratio of the probability of getting a composite number to the probability of getting a prime number is ............... .

  1. 2 : 3

  2. 3 : 2

  3. 1 : 3

  4. 1 : 2

When a die is cast with numbering on its faces, as shown, the ratio of the probability of getting a composite number to the probability of getting a prime number is . Maths Competency Focused Practice Questions Class 10 Solutions.

Answer

From numbers 1 to 6

Prime numbers = 2, 3, 5.

Composite numbers = 4, 6.

Probability of getting a prime number = No. of prime numbersNo. of possible outcomes=36=12\dfrac{\text{No. of prime numbers}}{\text{No. of possible outcomes}} = \dfrac{3}{6} = \dfrac{1}{2}.

Probability of getting a composite number

= No. of composite numbersNo. of possible outcomes=26=13\dfrac{\text{No. of composite numbers}}{\text{No. of possible outcomes}} = \dfrac{2}{6} = \dfrac{1}{3}.

Probability of getting a composite number to probability of getting a prime number = 13:12\dfrac{1}{3} : \dfrac{1}{2} = 2 : 3.

Hence, Option 1 is the correct option.

Question 46

The product of A = [1234]\begin{bmatrix*}[r] 1 & -2 \\ -3 & 4 \end{bmatrix*} and matrix M, AM = B where B = [224]\begin{bmatrix*}[r] 2 \\ 24 \end{bmatrix*}, then the order of matrix M is ............... .

  1. 2 × 2

  2. 2 × 1

  3. 1 × 2

  4. 4 × 1

Answer

Order of matrix A = 2 × 2

Order of matrix B = 2 × 1

Order of matrix M (let) = a × b

We know that,

Two matrix can be multiplied if the no. of columns of the first matrix is equal to the no. of rows of the second matrix and the resultant matrix has the no. of rows of first matrix and no. of columns of second matrix.

Given,

⇒ AM = B

⇒ A2 × 2 × Ma × b = B2 × 1

⇒ a = 2 and b = 1.

Order of matrix M = 2 × 1.

Hence, Option 2 is the correct option.

Question 47

Given, a1, a2, a3, ..... and b1, b2, b3, ..... are real numbers such that a1 - b1 = a2 - b2 = a3 - b3 = ......... are all equal.

a1 - b1, a2 - b2, a3 - b3, ........ forms a ......... progression.

  1. Geometric (r = 1)

  2. Arithmetic (d = 1)

  3. Geometric (r < 1)

  4. Arithmetic (d = 0)

Answer

Since,

a1 - b1 = a2 - b2 = a3 - b3.

∴ a1 - b1, a2 - b2, a3 - b3, ........ forms a arithmetic progression with common difference (d = 0).

Hence, Option 4 is the correct option.

Question 48

Locus of a moving point is ............... if it moves such that it keeps a fixed distance from a fixed point.

  1. Circle

  2. Line

  3. Angle

  4. Line segment

Answer

Locus of a moving point is circle if it moves such that it keeps a fixed distance from a fixed point.

Hence, Option 1 is the correct option.

Question 49

The point of concurrence of the angle bisectors of a triangle is called the ............... of the triangle.

  1. centroid

  2. incenter

  3. circumcenter

  4. orthocenter

Answer

The point of concurrence of the angle bisectors of a triangle is called the incenter of the triangle.

Hence, Option 2 is the correct option.

PrevNext